If orange glass is used in more of the windows than green glass, then the complete color combination of the glass in ...

ielkind on December 20, 2019

Why doesn't answer choice D work? I can't seem to figure that out

I placed Y,P, and R within window 1, G,P,O, and R within window 2, and O and G in window 3

Replies
Create a free account to read and take part in forum discussions.

Already have an account? log in

BenMingov on December 20, 2019

Hi ielkind, thanks for the question.

This is a tough game, so I understand your difficulty here.

It's important to keep in mind the strict numerical restriction this question forces upon us. In this case, it says more O than G. Meaning immediately it is either two or three Os are placed within the three windows.

Since, O and Y cannot be placed together, yet Y must be placed somewhere. This means that O will be restricted from at least one window. As a result, we can have O a maximum of twice.

If we have more O than G, then G can only be placed once. (I notice this is the mistake in the description of your setup. You have G placed twice).

Window 1: Y, P, R
Window 2: G, P, O, R
Window 3: P, O

This way we have more O (two) than G (one). P and O can be a complete window.

I hope this helps. Please let me know if you have any other questions.

mamie on December 26, 2019

I'm confused about the last rule. If not P -> O with the contrapositive of not O -> P. I took this rule as one window either has to have O OR P and cannot have both. Can you please clarify how this rule works so that one window can have both P and O. Thanks!

Skylar on January 5, 2020

@mamie, happy to help.

Your diagram of the last rule is correct. We have:
NOT P -> O
NOT O -> P

However, your interpretation of this diagram seems to be where you're getting confused. We know that if there is no P in a window, we must add O. We also know that if there is no O, we must add P. However, we know nothing if P is the Sufficient condition or if O is the Sufficient condition, meaning we have no deductions to make if we are first told that the window has P or if the window has O.

So, this diagram means that each window must have at least one of O or P, and could possibly have both. This can be remembered as "at least one is in."

Does that make sense? Please let us know if you have any other questions and best of luck with your studies!

shannonk68 on August 4, 2020

This answer makes no sense, at least not as far as the explanation video goes. It says in the video that O or P must appear in each window, but makes no reference to it possibly appearing together.

I feel like this question is faulty.

Skylar on August 4, 2020

@shannonk68, happy to help!

Rule #4 presents us with a classic Either/Or diagram.
NOT P -> O
NOT O -> P

Either/Or diagrams tell us that either the necessary condition in the original statement or the necessary condition in the contrapositive must be in. It also allows for the possibility that both are in. Let's think about this. If I tell you that you must study either Logic Games or Reading Comprehension today and you study both, have you met the criteria? Yes. Notice this is different from a situation in which I tell you that you must study either Logic Games or Reading Comprehension BUT NOT BOTH (BNB). Rule #4 does not specify that we cannot have both O and P in together, so that possibility is left open.

If you're looking for further explanation on this concept, I would point you to minute 59:12 of the Sufficient & Necessary Conditions Video and minute 2:58 of the Group Games video. Hope that helps!

Does that make sense? Please let us know if you have any other questions.

shannonk68 on August 5, 2020

so unless it specifically says "but not both" assume that they could be together, if it is seems like it's presenting it as an either/or rule?

Logic games are so illogical LOL

Thank you for your response.